Multivariable Chain Rule Question

Click For Summary
The discussion centers on applying the multivariable chain rule to the function f(x,y) = x/y, using polar coordinates with substitutions x = r cos(θ) and y = r sin(θ). A participant attempts to derive the partial derivative ∂f/∂x but encounters confusion regarding the correct application of the chain rule, particularly in how to treat the variables r and θ. The mistake identified is the incorrect assumption that both r and θ can be treated independently when taking the partial derivative with respect to x, leading to erroneous results. Clarification emphasizes that while taking partial derivatives, one must hold other variables constant, which resolves the misunderstanding. The discussion concludes with an acknowledgment of the error and appreciation for the guidance received.
Master1022
Messages
590
Reaction score
116
Homework Statement
Show that the result obtained can be done directly via the chain rule for partial derivatives.
Relevant Equations
Chain rule for partial derivatives
For context, we have an equation f(x,y) = \frac{x}{y} and we had used the substitutions x = r \cos\theta and y = r \sin\theta. In the previous parts of the question, we have shown the following result:
\frac{\partial f}{\partial x} = \cos\theta \Big(\frac{\partial f}{\partial r}\Big) - \frac{\sin\theta}{r} \Big(\frac{\partial f}{\partial \theta}\Big)

Now we have been asked to get to this result directly using the chain rule for partial derivatives.
My attempt: the chain rule states that:
\frac{\partial f}{\partial x} = \Big(\frac{\partial f}{\partial r}\Big) \Big(\frac{\partial r}{\partial x}\Big) + \Big(\frac{\partial f}{\partial \theta}\Big) \Big(\frac{\partial \theta}{\partial x}\Big)

This is where I seem to have gone wrong, and I would appreciate any insight as to why that may have been the case. The answers have used r^2 = x^2 + y^2 and \tan\theta = \frac{y}{x} and have gone on to the answer. I, however, just used the substitution above x = r\cos\theta, but this doesn't seem to lead us in the right direction. Showing my method, we get \frac{\partial r}{\partial x} = \frac{1}{cos\theta} and \frac{\partial \theta}{\partial x} = \frac{-1}{r\sin\theta}. Substituting into the above chain rule yields:

\frac{\partial f}{\partial x} = \Big(\frac{\partial f}{\partial r}\Big) \Big(\frac{1}{cos\theta} \Big) + \Big(\frac{\partial f}{\partial \theta}\Big) \Big(\frac{-1}{r\sin\theta}\Big). I am not sure I can see any clever way to get us to the right answer from here.

Looking back, I think the fact that I had two expressions for r in terms of x and y could have been part of the problem. However, I cannot seem to put an exact reason as to why that would be the case.

Thanks in advance.
 
Physics news on Phys.org
Please show your work in taking the derivatives.

Note that generally
$$
\frac{\partial f}{\partial g} \neq \frac{1}{\frac{\partial g}{\partial f}}.
$$
 
Orodruin said:
Please show your work in taking the derivatives.

Note that generally
$$
\frac{\partial f}{\partial g} \neq \frac{1}{\frac{\partial g}{\partial f}}.
$$

Thank you for your fast response. So my working was as follows:
r = \frac{x}{cos\theta} -------> \frac{\partial r}{\partial x} = \frac{1}{\cos\theta}
and for the theta/r relationship:
\cos\theta = \frac{x}{r} -------> \frac{\partial}{\partial x} (\cos\theta) = \frac{\partial}{\partial x} \Big(\frac{x}{r}\Big)
\frac{\partial}{\partial \theta} (\cos\theta) \Big(\frac{\partial \theta}{\partial x}\Big) = \frac{1}{r}\
\frac{\partial \theta}{\partial x} = \frac{-1}{r\sin\theta}
 
Master1022 said:
Thank you for your fast response. So my working was as follows:
r = \frac{x}{cos\theta} -------> \frac{\partial r}{\partial x} = \frac{1}{\cos\theta}
This is incorrect. When you are taking the partial derivative with respect to x, you are keeping y constant, not ##\theta##. Similar arguments for the other partial derivative.
 
  • Like
Likes Master1022
Orodruin said:
This is incorrect. When you are taking the partial derivative with respect to x, you are keeping y constant, not ##\theta##. Similar arguments for the other partial derivative.
Ahh yes, that makes sense. Many thanks for helping me out!
 
Question: A clock's minute hand has length 4 and its hour hand has length 3. What is the distance between the tips at the moment when it is increasing most rapidly?(Putnam Exam Question) Answer: Making assumption that both the hands moves at constant angular velocities, the answer is ## \sqrt{7} .## But don't you think this assumption is somewhat doubtful and wrong?

Similar threads

  • · Replies 3 ·
Replies
3
Views
3K
  • · Replies 1 ·
Replies
1
Views
1K
Replies
2
Views
1K
  • · Replies 8 ·
Replies
8
Views
2K
  • · Replies 5 ·
Replies
5
Views
1K
Replies
4
Views
1K
  • · Replies 3 ·
Replies
3
Views
2K
  • · Replies 11 ·
Replies
11
Views
2K
Replies
3
Views
2K
  • · Replies 4 ·
Replies
4
Views
2K